Standing on a bridge, you throw a stone straight upward. The stone hits a stream, 32.5 m below the point atwhich you release it, 3.10 s later. What is the speed of the stone (in m/s) just after it leaves your hand? Pleasedo not include any units in your answer below. Type in only the numerical result. If you include units, youranswer will be marked as incorrect.

Answers

Answer 1

Given:

Distance the stone hits the stream = 32.5m below the released point

Time = 3.10 seconds

Let's find the speed of the stone just after it leaves your hand.

To find the speed of the stone, apply the kinematic formula:

[tex]\Delta y=v_{iy}\ast t-\frac{1}{2}g\ast t^2[/tex]

Since the point the stone hits the stream is below the released point is, the change in distance is:

[tex]\Delta y=0-32.5=-32.5m[/tex]

Where:

a = -g = -9.8 m/s^2

t = 3.10 s

Substituet values into the formula and solve for the speed of the stone (vy).

We have:

[tex]\begin{gathered} -32.5=v_{iy}\ast3.10-\frac{1}{2}(9.8)\ast3.10^2 \\ \\ -32.5=v_{iy}\ast3.10-4.9\ast9.61 \\ \\ -32.5=v_{iy}\ast3.10-47.089 \end{gathered}[/tex]

Solving further:

[tex]\begin{gathered} v_{iy}\ast3.10=-32.5+47.089 \\ \\ v_{iy}\ast3.10=14.589 \end{gathered}[/tex]

Divide both sides by 3.10:

[tex]\begin{gathered} \frac{v_{iy}\ast3.10}{3.10}=\frac{14.589}{3.10} \\ \\ v_{iy}=4.706\text{ m/s} \end{gathered}[/tex]

Therefore, the speed of the stone just after it leaves your hand is 4.706 m/s

ANSWER:

4.706 m/s


Related Questions

A car with a mass of 1200kg is driving in circular path with radius of 65m at a constant speed of 5.5 m/s. What is the magnitude of the net force on the car? a) 102N b) 14182 N c) 6600 N d) 78000 N e) 558 N

Answers

Take into account that due to the speed of the car is constant, the net force on the car is the force due to the centrifugal force, which is given by the following formula:

[tex]F=ma_c=m\cdot\frac{v^2}{r}[/tex]

where:

m: mass of the car = 1200 kg

v: speed = 5.5 m/s

r: radius of the circular trajectory = 65 m

replace the previous values of the parameters into the formula for F:

[tex]F=1200kg\cdot\frac{(5.5m/s)^2}{65m}=558.46N\approx558N[/tex]

What happens to the strength of an electromagnet if the current in the wire is increased?

Answers

We will have the following:

What will happen is that the magnetic field will increase; since the magnetic field increases when the current in the wire of the electromagnet increases.

What are electromagnetic radiations?​

Answers

Radiations associated with electric and magnetic field is called Electromagnetic radiations.

electric and magnetic field radiations = electromagnetic radiations.

i need help please. i tried different answers but i cant get it.

Answers

Let's make a diagram to visualize the angle.

As you can observe, the angle is closer to the South orientation.

Therefore, the new angle is 42 degrees from South.

you have entered a 134-mile biathlon that consists of a run and a bicycle race. During your run, your average velocity is 6 miles per hour, and during your bicycle ace, your average velocity is 29 miles per hour. You finish the race in 7 hours. What is the distance of the run? What is the distance of the bicycle race?upposeWhat is the distance of the run?miles

Answers

ANSWER:

Distance of the run: 18 miles

Distance of the bicycle race: 116 miles

STEP-BY-STEP EXPLANATION:

Given:

Total distance = 134 miles

Total time = 7 hours

Average velocity during running = 6 mph

Average velocity during bicycle = 29 mph

Let x be the running distance and y be the bicycle distance.

We know that velocity equals distance in a given time, like this:

[tex]\begin{gathered} v=\frac{d}{t} \\ \\ \text{ Therefore:} \\ \\ t=\frac{d}{v} \end{gathered}[/tex]

Knowing the above, we can establish the following system of equations:

[tex]\begin{gathered} t_1+t_2=7\rightarrow\frac{d_1}{v_1}+\frac{d_2}{v_2}=7\rightarrow\frac{x}{6}+\frac{y}{29}=7\text{ \lparen1\rparen} \\ \\ x+y=134\rightarrow x=134-y\text{ \lparen2\rparen} \end{gathered}[/tex]

We substitute the second equation in the first and obtain the following:

[tex]\begin{gathered} \frac{134-y}{6}+\frac{y}{29}=7 \\ \\ \frac{(134-y)(29)+6y}{6\cdot29}=7 \\ \\ \frac{3886-29y+6y}{174}=7 \\ \\ 3886-23y=7\cdot174 \\ \\ y=\frac{1218-3886}{-23}=\frac{-2668}{-23} \\ \\ y=116\rightarrow\text{ bicycle distance} \\ \\ \text{ now, for x:} \\ \\ x=134-116 \\ \\ x=18\rightarrow\text{ running distance} \end{gathered}[/tex]

Therefore:

The distance of running is 18 miles and the distance by bicycle is 116 miles.

An object moving with uniform acceleration has a velocity of 13.0 cm/s in the positive x-direction when its x-coordinate is 2.76 cm. If its x-coordinate 3.05 s later is −5.00 cm, what is its acceleration?______ cm/s2

Answers

Since the object is moving with uniform acceleration we have an uniformly accelerated motion which means that we can use the following equations:

[tex]\begin{gathered} a=\frac{v_f-v_0}{t} \\ x=x_0+v_0t+\frac{1}{2}at^2 \\ v_f^2-v_0^2=2a(x-x_0) \end{gathered}[/tex]

Now, in this case we know:

• The initial position 2.76 cm.

,

• The initial velocity 13 cm/s

,

• The final position -5 cm

,

• The time it takes 3.05 s.

And we want to determine the acceleration; from what we know and what we want we determine that we can use the second equation. Plugging the values in that equation we have that:

[tex]\begin{gathered} -5=2.76+(13)(3.05)+\frac{1}{2}(3.05)^2a \\ \frac{3.05^2}{2}a=-5-2.76-(13)(3.05) \\ \frac{3.05^2}{2}a=-47.41 \\ 3.05^2a=-94.82 \\ a=-\frac{94.82}{3.05^2} \\ a=-10.19 \end{gathered}[/tex]

Therefore, the acceleration is -10.19 cm/s²

General relativity is combines special relativity with the equivalence principle.Why and why not?

Answers

The equivalence principle is a fundamental law of physics that states that inertial forces and gravitational forces are similar in nature and are often indistinguishable from each other.

The application of the equivalence principle to in combination with the general theory of relativity allowed the refinement of the equations of the theory helping give birth to the theory of general relativity.

For example, in special relativity objects are considered to be moving with a constant velocity. When the principle of equivalence is introduced this allows considering inertial frames or accelerated frames of reference and the gravitational forces in a system thus completing the special theory of relativity.

When the acceleration in the general theory of relativity is considered to be zero then it reduces to the special theory of relativity.

A ballistic pendulum consists of a 1.25-kg block of wood that is hanging from the ceiling in such a way that when a bullet enters it, the block’s change in height can be recorded as it swings. A bullet having a mass of 6.25-grams and unknown velocity strikes the block and becomes imbedded in it. The impulse imparted to the block causes it to swing in such a way that its height increases by 7.15 cm.1. What was the change in potential energy of the block/bullet combo after the collision?2. What was the speed of the block/bullet combo immediately after the collision (and before it beganto swing)?3. What was the speed of the bullet before entering the block of wood?

Answers

1.

The potential energy is defined as:

[tex]U=mgh[/tex]

The change in potential energy is:

[tex]\Delta U=U_f-U_i[/tex]

the initial height of the block/bullet is zero, then we have that:

[tex]\begin{gathered} \Delta U=(1.25+0.00625)(9.8)(0.0715)-0 \\ \Delta U=0.8802 \end{gathered}[/tex]

Therefore, the change in potential energy is 0.8802 J

2.

We know that the energy is conserved, this means that the kinetic energy inmediately after collision has to be equal to the potential energy at the 7.15 cm heigh, then we have:

[tex]\begin{gathered} \frac{1}{2}mv^2=0.8802 \\ \frac{1}{2}(1.25625)v^2=0.8802 \\ v=\sqrt[]{\frac{2\cdot0.8802}{1.25625}} \\ v=1.184 \end{gathered}[/tex]

Therefore, the velocity at this moment is 1.184 m/s

3.

From conservation of momentum we know that:

[tex]m_bv_b=mv[/tex]

then:

[tex]\begin{gathered} (0.00625)v_b=(1.25625)(1.184) \\ v_b=\frac{(1.25625)(1.184)}{0.00625} \\ v_b=237.98 \end{gathered}[/tex]

Therefore the speed of the bullet is 237.98 m/s

Part C and D please, part A=291.2 and part B=33.8

Answers

Given,

The mass of the skater, m=63 kg

The coefficient of static friction, μs=0.4

The coefficient of the kinetic friction, μk=0.02

F₁=242 N

F₂=162 N

(c) The static friction is given by,

[tex]f_s=N\mu_s[/tex]

Where N is the normal force.

The normal force acting on the skater is

[tex]N=mg[/tex]

Where g is the acceleration due to gravity.

Therefore the static friction is given by,

[tex]f_s=mg\mu_s[/tex]

On substituting the known values,

[tex]\begin{gathered} f_s=63\times9.8\times0.4 \\ =246.96\text{ N} \end{gathered}[/tex]

Therefore the static friction on the skater is 246.96 N

d)The net force acting on the skater is

[tex]\begin{gathered} F_{\text{net}}=ma_{} \\ =F_{\text{tot}}-f \\ =F_{\text{tot}}-N\mu_k \\ =F_{\text{tot}}-mg\mu_k \end{gathered}[/tex]

On substituting the known values,

[tex]\begin{gathered} 63a=291.2-63\times9.8\times0.02 \\ a=\frac{278.85}{63} \\ =4.43m/s^2 \end{gathered}[/tex]

Thus the acceleration of the skater is 4.43 m/s²

How much does James have to pay for a vaccum cleaner that he uses for 4 hours a week.  In James's neighborhood, the cost of electricity is 27 cents per kwhr.The power is P = 800 W

Answers

Given:

The power of the vacuum cleaner is P = 800 W = 0.8 kW

The time consumption is t = 4 hours a week.

The cost of electricity is 27 cents per kWh

To find the cost of the electricity for using the vacuum cleaner for a week.

Explanation:

First, we need to find the energy.

The energy can be calculated as

[tex]\begin{gathered} E=P\times t \\ =0.8\times4 \\ =3.2\text{ kWh} \end{gathered}[/tex]

The cost of electricity per week due to vacuum cleaner will be

[tex]27\times3.2=86.4\text{ cents}[/tex]

Final Answer: The cost of electricity James has to pay for a vacuum cleaner for using it 4 hours a week is 86.4 cents.

in four hours, a hiker in a canyon goes from 892ft to 256 ft above the canyon floor. Find the hikers vertical speed.

Answers

From the given question, we can deduce the following information:

• Time, t = 4 hours

,

• The hiker goes from 892 ft to 256 ft above the canyon floor.

Let's find the vertical speed of the hiker.

To find the speed, apply the formula:

[tex]speed=\frac{dis\tan ce}{time}[/tex]

To find the distance, we have:

Distance covered = 256 ft - 892 ft = -636 ft.

Hence, to find the vertical speed, we have:

[tex]\begin{gathered} speed=\frac{dis\tan ce}{time} \\ \\ \text{speed}=\frac{-636}{4}=-159\text{ ft/h} \\ \end{gathered}[/tex]

Therefore, the vertical speed of the hiker is -159 feet per hour.

ANSWER:

Victor is driving south. He is traveling at 12 m/s, when he enters an area with a new speed limit. Aver a period of 6 seconds, his speed increases from 12 m/s to 29 m/s. What is Victor’s acceleration during this period?

Answers

Answer:

1.1417 m/s²

Explanation:

Acceleration = final velocity - initial velocity/ time

a= [tex]\frac{v-u}{t}[/tex]

a= [tex]\frac{29-12}{12}[/tex]

a= [tex]\frac{17}{12}[/tex]

a= 1.417 m/s²   or [tex]\frac{17}{12}[/tex] m/s² in fraction form

Two figure skaters, one weighing 625 N and the other 725 N, push off against each other onfrictionless ice. If the heavier skater travels at 1.5 m/s, how fast will the lighter one travel?A 1.7 m/sB 2.8 m/sC -1.7 m/sD -2.8 m/s

Answers

We are given the following information:

Weight of skater 1 = 625 N

Weight of skater 2 = 725 N

Final velocity of skater 2 = 1.5 m/s

Final velocity of skater 1 = ?

Recall from the law of conservation of momentum, the total momentum before the collision and after the collision must be equal.

[tex]\begin{gathered} p_{before}=p_{after} \\ m_1u_1+m_2u_2=m_1v_1+m_2v_2 \end{gathered}[/tex]

The initial velocities of both skaters are 0 m/s

[tex]m_1\cdot0_{}+m_2\cdot0=m_1v_1+m_2v_2[/tex]

Also, m = W/g

[tex]\begin{gathered} 0=m_1v_1+m_2v_2 \\ 0=(\frac{625}{9.8})\cdot_{}v_1+(\frac{725}{9.8})\cdot1.5 \\ (\frac{625}{9.8})\cdot_{}v_1=-(\frac{725}{9.8})\cdot1.5 \\ (63.78)\cdot_{}v_1=-110.97 \\ _{}v_1=-\frac{110.97}{63.78} \\ _{}v_1=-1.7\: \frac{m}{s} \end{gathered}[/tex]

So, the lighter skater will travel with a velocity of 1.7 m/s

The negative sign means that the lighter skater will be traveling oppositely to the heavier skater.

I have some problems applying the formulas to solve physics problems. I understand all the concepts needed, but just freeze when I see questions, especially when it comes to trying to combine linear and rotational conceptsA uniform, 255 N rod that is 1.90 m long carries a 225 N weight at its right end and an unknown weight W toward the left end (Figure 1). When W is placed 60.0 cm from the left end of the rod, the system just balances horizontally when the fulcrum is located 75.0 cm from the right end.1) Find W.2) If W is now moved 30.0 cm to the right, how far must the fulcrum be moved to restore balance?

Answers

ANSWER

[tex]\begin{gathered} 1)\text{ }214.90\text{ }N \\ \\ 2)\text{ }0.09\text{ }m \end{gathered}[/tex]

EXPLANATION

First, let us make a sketch of the diagram showing the distances on the rod:

1) Since the fulcrum is balanced, the center of gravity of the system will be at the fulcrum.

The center of gravity (in the horizontal is given by:

[tex]x=\frac{W_1x_1+W_2x_2+W_3x_3}{W_1+W_2+W_3}[/tex]

where W1 = the weight on the right end = 225 N

W2 = the weight of the rod = 255 N

W3 = the weight place on the left = W

x1 = the position of W1 (taking the left as the origin) = 1.90 m

x2 = the position of the center of mass of the rod = x1/2 = 0.95 m

x3 = the position of W from the left end = 0.60 m

x = position of center of gravity of the rod from the left end i.e. at the fulcrum = 1.90 - 0.75 = 1.15 m

Now, substitute the values given in the question and solve for W:

[tex]\begin{gathered} 1.15=\frac{(225*1.90)+(255*0.95)+(W*0.60)}{225+255+W} \\ \\ 1.15=\frac{427.5+242.25+0.60W}{480+W} \\ \\ 1.15(480+W)=669.75+0.60W \\ \\ 552+1.15W=669.75+0.60W \\ \\ 1.15W-0.60W=669.75-552 \\ \\ 0.55W=117.75 \\ \\ W=\frac{117.75}{0.55} \\ \\ W=214.09\text{ }N \end{gathered}[/tex]

That is the value of W.

2) Now, W is moved 30.0 cm (0.30 m) to the right.

This implies that:

[tex]x_3=0.60+0.30=0.90\text{ }m[/tex]

Since the other values (including W) do not change, we can now solve for x, which is the new center of gravity:

[tex]\begin{gathered} x=\frac{(225\times1.90)+(255\times0.95)+(214.09\times0.90)}{225+255+214.09} \\ \\ x=\frac{427.5+242.25+192.681}{694.09}=\frac{862.431}{694.09} \\ \\ x=1.24\text{ }m \end{gathered}[/tex]

Therefore, the fulcrum must be moved:

[tex]\begin{gathered} 1.24\text{ }m-1.15\text{ }m \\ \\ 0.09\text{ }m \end{gathered}[/tex]

The fulcrum should be moved 0.09 m to the right (since the W is moved to the right).

Calculate the torque experienced by the door due to this force using torque is equal to force times lever armLength of the lever arm = 1 mForce = 5 N

Answers

Given:

The applied force on the door is F = 5 N

The length of the lever arm is l = 1 m

Required: Torque experienced by the torque.

Explanation:

Torque is the product of force and the distance between the force applied and the rotational axis.

The force is applied on one side of the lever arm while the rotational axis will be at the other end of the lever arm.

So, the distance between the rotational axis and the force applied is the length of the arm.

Torque can be calculated by the formula

[tex]\tau=F\times l[/tex]

On substituting the values, the torque will be

[tex]\begin{gathered} \tau=5\times1 \\ =5\text{ N m } \end{gathered}[/tex]

Final Answer: The torque experienced by the door is 5 N m.

which choice are equivalent to the expression below? check all that apply. 4 square root 5answer choices: square root of 16*6, square root of 32*3, square root of 96, square of 24, 96, square root of 4*36

Answers

Given

4 square root 6

[tex]4\sqrt[]{6}[/tex]

Wich choices are equivalent to the expression below

[tex]\begin{gathered} \sqrt[]{16}\sqrt[]{6} \\ \sqrt[]{32}\sqrt[]{3} \\ \sqrt[]{96} \end{gathered}[/tex]

The first three options are the correct answers.

Pascal's principle states that:all liquids exert pressure downward.pressure will move toward areas where pressure is lowest.applied pressure will be transmitted throughout a fluid.None of the choices are correct.

Answers

According to Pascal's principle states that applied pressure will be transmitted throughout a fluid.

Thus, third one is the correct option.

how do I convert fractions into percentages like 14 over 20

Answers

Given data

*The given fraction is

[tex]x=\frac{14}{20}[/tex]

Convert the given fraction into percentage as

[tex]\begin{gathered} x=\frac{14}{20}\times100\text{\%} \\ =0.7\times100\text{\%} \\ =70\text{\%} \end{gathered}[/tex]

The fundamental frequency of a pipe is the lowest resonant frequency. Is this true or false?

Answers

By definition, the fundamental frequency of an object is the lowest frequency such that the object resonates.

Then, the answer is:

[tex]\text{True}[/tex]

Answer:

By definition, the fundamental frequency of an object is the lowest frequency such that the object resonates.

Then, the answer is: true

Explanation:

What is the maximum speed at which a cyclist can move on a bend and at an angle from the vertical, he should deviate to the side of the bend, so as not to fall, if the coefficient of friction of the wheel from the road is 0.4 and the radius of curvature of the road is 100m

Answers

Given,

The coefficient of friction between the wheel and the road, μ=0.4

The radius of curvature of the road, r=100 m

The centripetal force for the cyclist to move in the curved path is provided by the friction between the road and the wheel.

Thus,

[tex]\begin{gathered} \frac{mv^2}{r}=\mu mg \\ \Rightarrow v^2=r\mu g \\ v=\sqrt[]{r\mu g} \end{gathered}[/tex]

Where m is the mass of the cyclist and the cycle and g is the acceleration due to gravity.

The solar system formed about 4.5 billion years ago. This expressed asA. 4.5x10^6 yearsB. 4.5x10^7 yearsC. 4.5x10^8 yearsD. 4.5 x10^9 years

Answers

Given,

The age of the solar system is 4.5 billion years.

One billion is 1,000,000,000.

In scientific notation, we can write one billion as 1×10⁹.

Therefore to express any number in billions we need to multiply it with the above number.

Therefore, we can write 4.5 billion years as,

[tex]4.5\times1\times10^9=4.5\times10^9\text{ years}[/tex]

Therefore we can express 4.5 billion years as 4.5

John is at the park playing fetch with his cat. He throws the ball 1m south, and his cat retrieves the ball and returns it to John. John then throws the ball 3 meters, and the cat again collects the ball and returns it to John. When the cat returns for the second time, what distance has the cat traveled?6m8m2m4m

Answers

As John throws the ball 3 meters and the cat collects the ball and returns to John. Then. the distance traveled by the cat when it returns from the second time is calculated as

[tex]\begin{gathered} d=3+3 \\ =6\text{ m} \end{gathered}[/tex]

what happens to F 1. m is doubled2. m is tripled3. m an m are both doubled4. m is halved5. r is doubled6. r is tripled7. r is increased 10 times8. r is halved

Answers

The expression for the gravitational force is given as:

[tex]F_g=\frac{GMm}{r^2}[/tex]

1. When M is doubled, then force will also be doubled as Force is directly proportional to the mass.

[tex]\begin{gathered} F\alpha M \\ F\alpha m \\ F\alpha\frac{1}{r^2} \end{gathered}[/tex]

2. When the m is tripled, then force will be tripled.

[tex]\begin{gathered} F^{\prime}\alpha\frac{M\times3m}{r^2} \\ F^{\prime}\alpha\frac{3Mm}{r^2} \\ F^{\prime}\alpha3F \end{gathered}[/tex]

3. When both M and m are doubled then, the force will become four times.

[tex]\begin{gathered} F^{\prime}\alpha\frac{2M\times2m}{r^2} \\ F^{\prime}\alpha4F \end{gathered}[/tex]

4.When m is halved, then Force will be halved, as force is directly proportional to the mass.

[tex]\begin{gathered} F^{\prime}\alpha\frac{M\times\frac{m}{3}}{r^2} \\ F^{\prime}\alpha\frac{Mm}{3r^2} \\ F^{\prime}\alpha\frac{F}{3} \end{gathered}[/tex]

5. When distance is doubled, then force will become one-fouth.

[tex]\begin{gathered} F^{\prime}\alpha\frac{Mm}{(2r)^2} \\ F^{\prime}\alpha\frac{Mm}{r^2}\times\frac{1}{4} \\ F^{\prime}\alpha\frac{F}{4} \end{gathered}[/tex]

6.When the distance is tripled, the force will become one-ninth.

[tex]\begin{gathered} F^{\prime}\alpha\frac{Mm}{(3r)^2} \\ F^{\prime}\alpha\frac{Mm}{r^2}\times\frac{1}{9} \\ F^{\prime}\alpha\frac{F}{9} \end{gathered}[/tex]

7. When the distance is increased to ten times, the force will become one-hundredth.

[tex]\begin{gathered} F^{\prime}\alpha\frac{Mm}{(10r)^2} \\ F^{\prime}\alpha\frac{Mm}{r^2}\times\frac{1}{100} \\ F^{\prime}\alpha\frac{F}{100} \\ 8.\text{ When the distance in halved, the force will be four time.} \\ F^{\prime}\alpha\frac{Mm}{(\frac{r}{2})^2} \\ F^{\prime}\alpha4\frac{Mm}{r^2} \\ F^{\prime}\alpha4F \end{gathered}[/tex]

Find the y-component of thisvector:12.0 m73.3°Remember, angles are measured fromthe +X axis.y-component (m)

Answers

Given data:

The magnitude of the given vector is,

[tex]A=12.0\text{ m}[/tex]

An angle between the given vector and +x axis is,

[tex]\theta=73.3^o[/tex]

The formula of y-component is as follows:

[tex]A_y=A\sin \theta[/tex]

Here,

[tex]A\text{ is the magnitude of the given vector-A}[/tex]

Now, substitute known values in above equation;

[tex]\begin{gathered} A_y=12\text{ m}\times\sin 73.3^o \\ A_y=11.49\text{ m} \end{gathered}[/tex]

Therefore, the y-component of the given vector is 11.49 m

The mass of a satellite orbiting Earth is 15000 kg.

Answers

Newton's universal law of gravity

[tex]F=G\frac{m1\cdot m2}{r^2^{}}[/tex]

Where:

F= force between objects

m1= mass 1 = 15,000 kg

m2= mass 2 = 6x10^24

r = distance = 34,000,000 m

G= universal contant of gravitation = 6.67 x10^-11

Replacing:

[tex]F=6.67\cdot10^{-11}\cdot\frac{15,000\cdot6x10^{24}}{(34,000,000)^2}[/tex]

F= 5,193 N

One way to create more fissionable fuel in the form of plutonium-239 is in which of the following?Select one:a.a coal-fire power station b.radioactive wastec.a meltdown of the cored.a breeder reactor

Answers

The correct answer is radioactive waste.

The Plutonium-239 is waste material of fuel road used in a nuclear power plant for the electricity production. The use of nuclear power plant will increase the radioactive waste and the production of Plutonum-239 will be more.

Thus, option b is

URGENT!! ILL GIVE
BRAINLIEST!!!! AND 100 POINTS!!!!!!

Answers

Answer:

b heated gas will have decreased kinetic energy and decreased density

Explanation:

gas loses weight

You're standing on a ramp that is inclined at a 25 degree angle. The weight (Fg) experienced is 50N. What is the parallel force (Fx) and the perpendicular force (Fy)?I need to do the following: 1. Draw a free body diagram2.Identify Givens and Unknowns3.Identify the Equations4.Set up the equation using the givens and unknowns5.Solve

Answers

Answer:

Fx = 21.13 N

Fy = 45.32 N

Explanation:

In this case, the free body diagram is:

We can see that there is a right triangle formed by the weight (Fg) and the ramp. So, we can calculate the value of angle θ.

25 + 90 + θ = 180

115 + θ = 180

θ = 180 - 115

θ = 65

Because the sum of the interior angles of a triangle is always 180 degrees.

Now, we can calculate the parallel force and the perpendicular force using the trigonometric functions sine and cosine, so

Fx = Fg cos θ

Fy = Fg sin θ

So, replacing Fg = 50 N and θ = 65, we get:

Fx = 50 cos(65) = 21.13 N

Fy = 50 sin(65) = 45.32 N

Therefore, the answers are:

Fx = 21.13 N

Fy = 45.32 N

According to Coulomb's law, the electrical force between two charged objects:A.is zero if they are opposite charges.B.increases with increasing charge.C.does not depend on the amount of charge.D.increases with increasing distance.

Answers

Answer and explanation:

A correct option is an option (B).

The electrical force between two charges is given as,

[tex]F=\frac{1}{4\pi\epsilon_0}\frac{q_1q_1}{r^2}[/tex]

The electrical force is directly proportional to the product of two charges. Thus Force will depend on two charges irrespective of their signs.

Option (A) is incorrect because if charges are opposite, the value of force will not be zero. It will be -ve.

Option (C) is incorrect because the force is directly proportional to the product of charges, it depends on the amount of charge.

Option (D) is also incorrect because the force in inversly proportional to the distance between two charges. Thus, if the distance between two charges is increased, the force between two charges will decrease.

Concllusion:

The correct option is option (B).

A guitar player tunes her strings so thatthere is a beat frequency of 1.0 Hzbetween them. If one string has afrequency of 220 Hz, what is thefrequency of the other string? (Thereare two possible answers; give one.) -(Unit = Hz)

Answers

Fb = beat frequency = 1 Hz

F1 = frequency 1 = 220 Hz

Fb = l F1 - F2 l or

Fb= l F2 - F1 l

Replacing:

1 = 220 - F2

f2 = 220 - 1

f2 = 219 HZ

Other Questions
In a compound that contains monatomic ions, which of the following gets named first?the nonmetalthe metalthe anionthe roman numeral of the valence number Two points determine a line. Find an equation of the line passing through the points.(2,7) and (5,19)An equation of the line is y= Two astronauts, of masses 60 kg and 80 kg, are initially right next to each other and at rest in outer space. They suddenly push each other apart. What is their separation after the heavier astronaut has moved 12 m?24 m16 m9.0 m28 m21 m Point M is located at (-4,-6)What is located 4units from point M? Use the information from the previous question to answer this question now that you know the length of the missing side of the triangle, find the actual distance. Since each unit on the grid is zero. 5 mile, the ferry will travel about blank miles in a class, 2/3 of the pupils are boys. If there are 15 more boys than girls, how many pupils are there in the class (use equation method) Determine if the two triangles are congruent. If they are, complete the congruence statement; and identify how they are congruent. Otherwise, select "Not Congruent" for BOTH drop downs.Be sure to identify any corresponding angles or sides that you know are congruent!AniBby[ Select ]AACB ZA [Select ] The volume of a cube is 2744m^3. What is the length of an edge of the cube? can you help me with the fourth one marked as e Carlos is not sure whether he should use a normal adult-size blood pressure cuff or a large adultcuff. How would he be able to tell if he needs the large adult cuff? Why is this important? Two 4.952 cm by 4.952 cm plates that form a parallel-plate capacitor are charged to +/- 0.576 nC. What is the electric field strength inside the capacitor if the spacing between the plates is 2.97 mm? Due 05/05/200:00Tickets to a school play cost $3 for students and $8 for adults. On opening night, $1,000 was collected and 150 tickets sold Use substitution to solvea system of equations to find how many of each kind of ticket were sold? Enter your answers in the boxesstudent ticketsadult tickets 7 While grocery shopping on a Sunday afternoon, Jonas sees a pot roast that has been marked down. The pot roast has an expiration date set for Tuesday, but Jonas wants to host dinner for friends on Friday night. Which would be the BEST way for Jonas to make sure the pot roast is safely prepared? A. Cook the pot roast on Sunday and refrigerate it until Friday night. B. Buy the pot roast and keep it in the refrigerator until Friday night. C. Freeze the pot roast until he is ready to prepare it Friday night. D. Keep the pot roast in the refrigerator but cook it on Tuesday. What features should you present when demonstrating 2023 armadas confident handling?. Hi so sorry i need help in geometry i think its asking if the two given angles are coterminal or not Need help. Not understanding how to start with each of these Which of the following functions best models the data shown in the scatterplot below? y=3x+5y=x2+10y=x+8y=2x+ Find the area of the shaded region in the figure. Use the pi key for pi.The area of the shaded region in the figure is approximately: (Type an integer or decimal rounded to the nearest tenth as needed.) Hi can you help me with this question real quick please Use the graph of the function to estimate the interval on which the function is decreasing.Enter your answer in interval notation.Enter any values to one decimal place.